LSAT and Law School Admissions Forum

Get expert LSAT preparation and law school admissions advice from PowerScore Test Preparation.

 LSAT2018
  • Posts: 242
  • Joined: Jan 10, 2018
|
#45415
Would the part 'given that a price that will prove to be right is virtually impossible for the seller to gauge in advance' in the final sentence be an intermediate conclusion?
 lathlee
  • Posts: 652
  • Joined: Apr 01, 2016
|
#58318
This is a broad from of Resolve the Pardox or Point at issue?
 Jay Donnell
PowerScore Staff
  • PowerScore Staff
  • Posts: 144
  • Joined: Jan 09, 2019
|
#61778
Hi Lathlee and LSAT2018!


First, for Lathlee, this question would be considered a Method of Reasoning question. That particular type can be extremely variant by the stem, and in this particular case the answer choice language is also helpful to decipher what we are being asked to do. Here, the answer helps explain a part of what the argument is doing, so we are helping to explain the method of reasoning used by the author. I find a key word to remember in these questions is the word HOW, as in how does the author do what they've done? I hope that helps!

For LSAT2018, I do feel that this argument includes a subsidiary conclusion, but I don't think it's that particular claim. Statements preceded by 'given that' can be read as premises, as we are given that information to trust without any supporting statements to back it up. The first half of the second sentence is bracketed between an 'in fact' and a 'since' after a dividing comma, which help me reason that the statement about selling too low being in serious error is supported by other claims, so it serves as a conclusion by definition. That idea helps to support the argument's main conclusion at the bottom advising that it's best to err on the high side for pricing.

I hope that was helpful to you both!
 yournoona
  • Posts: 18
  • Joined: Mar 13, 2020
|
#74545
Hello.
I am having trouble understanding why is the answer A instead of D.
As I understood the question,
Since exclusive properties sell quickly even though expensive and
if counterstrategy i.e. low to high charge applied → exclusivity in question.
CONCLUSION:
The seller cant predict the high charge in advance so any price should be high from the start.
Option D focuses clearly on the conclusion by saying that the strategy doesn't depend on buyers estimates since the seller initially pegs it at a high cost.
And option D focuses on a counterproductive feature of the rejected alternative, if so that is also true considering the premise that the buyer would call into the question the exclusivity of the product.
 Jeremy Press
PowerScore Staff
  • PowerScore Staff
  • Posts: 1000
  • Joined: Jun 12, 2017
|
#74566
Hi yournoona,

Answer choice D is incorrect because it doesn't match what the argument states as its premise (its "grounds," to use the term from the question stem). What the argument states as its premise is this: "given that a price that will prove to be right is virtually impossible for the seller to gauge in advance." Notice that premise focuses on the difficulty (indeed, the impossibility) for the seller of gauging the price in advance of selling the item. What answer choice D refers to is the "prospective buyers' estimates." But the premise of the argument in the stimulus itself says nothing about the prospective buyers. In fact, it explicitly refers to the seller (and the difficulty the seller has in determining the right price). Thus, answer choice D is an inaccurate description of what the argument explicitly states and must be ruled out. Just remember, on a method of reasoning question, the description in the answer choice has to be matched precisely with the portion of the argument it describes.

I hope this helps!

Jeremy
 yournoona
  • Posts: 18
  • Joined: Mar 13, 2020
|
#74708
Yes,it helps. Thank you so much :-D
 w35t
  • Posts: 6
  • Joined: Jun 22, 2020
|
#77287
Could someone explain why answer choice A is correct?

Thank you
 Adam Tyson
PowerScore Staff
  • PowerScore Staff
  • Posts: 5153
  • Joined: Apr 14, 2011
|
#77646
Happy to, w35t! The author argues that when pricing an exclusive item for sale, the seller should err on the side of overpricing it rather than underpricing it. His reason is that underpricing it will weaken the "exclusive" appeal of the product, and overpricing it does not have that problem. Thus, the author's grounds are that his strategy is better because it "lacks a counterproductive feature" that comes with the strategy of underpricing. Overpricing won't have the same negative impact that underpricing would have.
 kupwarriors9
  • Posts: 73
  • Joined: Jul 01, 2021
|
#89022
I don't get how "this strategy lacks a counterproductive feature of the rejected alternative" relates to the strategy of underpricing. Overpricing won't have the same negative impact that overpricing would have." Is there any chance you could explain? Thank you
Adam Tyson wrote: Sun Aug 02, 2020 11:27 am Happy to, w35t! The author argues that when pricing an exclusive item for sale, the seller should err on the side of overpricing it rather than underpricing it. His reason is that underpricing it will weaken the "exclusive" appeal of the product, and overpricing it does not have that problem. Thus, the author's grounds are that his strategy is better because it "lacks a counterproductive feature" that comes with the strategy of underpricing. Overpricing won't have the same negative impact that overpricing would have.
 Adam Tyson
PowerScore Staff
  • PowerScore Staff
  • Posts: 5153
  • Joined: Apr 14, 2011
|
#89027
Break it down piece by piece, kupwarriors9, like this:

The rejected alternative is underpricing.

The counterproductive feature of underpricing is weakening the exclusive appeal.

Overpricing does not have that feature . It does not weaken the exclusive appeal.

So, this strategy (overpricing) lacks (does not have) a counterproductive feature (weakening the exclusive appeal) of the rejected alternative (underpricing). What I meant to say in my earlier post was "Overpricing won't have the same negative impact that underpricing would have." I'll fix that typo now!

Get the most out of your LSAT Prep Plus subscription.

Analyze and track your performance with our Testing and Analytics Package.